Difference between revisions of "1983 AHSME Problems/Problem 20"

(Solution)
(Fixed formatting and improved readability of the solution)
Line 4: Line 4:
 
are the roots of <math>x^2 - rx + s = 0</math>, then <math>rs</math> is necessarily  
 
are the roots of <math>x^2 - rx + s = 0</math>, then <math>rs</math> is necessarily  
  
<math>\text{(A)} \ pq \qquad  
+
<math>\textbf{(A)} \ pq \qquad  
\text{(B)} \ \frac{1}{pq} \qquad  
+
\textbf{(B)} \ \frac{1}{pq} \qquad  
\text{(C)} \ \frac{p}{q^2} \qquad  
+
\textbf{(C)} \ \frac{p}{q^2} \qquad  
\text{(D)}\ \frac{q}{p^2}\qquad
+
\textbf{(D)}\ \frac{q}{p^2}\qquad
\text{(E)}\ \frac{p}{q}</math>     
+
\textbf{(E)}\ \frac{p}{q}</math>     
  
 
== Solution ==
 
== Solution ==
By Vieta's Formulas, we have <math>\tan(\alpha)\tan(\beta)=q</math> and <math>\cot(\alpha)\cot(\beta)=s</math>. Recalling that <math>\cot\theta=\frac{1}{\tan\theta}</math>, we have <math>\frac{1}{\tan(\alpha)\tan(\beta)}=\frac{1}{q}=s</math>.  
+
By Vieta's Formulae, we have <math>\tan(\alpha)\tan(\beta)=q</math> and <math>\cot(\alpha)\cot(\beta)=s</math>. Recalling that <math>\cot\theta=\frac{1}{\tan\theta}</math>, we have <math>\frac{1}{\tan(\alpha)\tan(\beta)}=\frac{1}{q}=s</math>.  
  
By Vieta's Formulas, we have <math>\tan(\alpha)+\tan(\beta)=p</math> and <math>\cot(\alpha)+\cot(\beta)=r</math>. Recalling that <math>\cot\theta=\frac{1}{\tan\theta}</math>, we have <math>\tan(\alpha)+\tan(\beta)=r(\tan(\alpha)\tan(\beta))</math>. Using <math>\tan(\alpha)\tan(\beta)=q</math> and <math>\tan(\alpha)+\tan(\beta)=p</math>, we get that <math>r=\frac{p}{q}</math>, which yields a product of <math>\frac{1}{q}\cdot\frac{p}{q}=\frac{p}{q^2}</math>.
+
Also by Vieta's Formulae, we have <math>\tan(\alpha)+\tan(\beta)=p</math> and <math>\cot(\alpha)+\cot(\beta)=r</math>, and again using <math>\cot\theta=\frac{1}{\tan\theta}</math>, we have <math>\tan(\alpha)+\tan(\beta)=r(\tan(\alpha)\tan(\beta))</math>. Using <math>\tan(\alpha)\tan(\beta)=q</math> and <math>\tan(\alpha)+\tan(\beta)=p</math>, we therefore deduce that <math>r=\frac{p}{q}</math>, which yields <math>rs = \frac{1}{q}\cdot\frac{p}{q}=\frac{p}{q^2}</math>.
  
Thus, the answer is <math>(\text{C}) \ \frac{p}{q^2}</math>
+
Thus, the answer is <math>\boxed{\textbf{(C)} \ \frac{p}{q^2}}</math>.
 
==See Also==
 
==See Also==
 
{{AHSME box|year=1983|num-b=19|num-a=21}}
 
{{AHSME box|year=1983|num-b=19|num-a=21}}
  
 
{{MAA Notice}}
 
{{MAA Notice}}

Revision as of 17:04, 27 January 2019

Problem 20

If $\tan{\alpha}$ and $\tan{\beta}$ are the roots of $x^2 - px + q = 0$, and $\cot{\alpha}$ and $\cot{\beta}$ are the roots of $x^2 - rx + s = 0$, then $rs$ is necessarily

$\textbf{(A)} \ pq \qquad  \textbf{(B)} \ \frac{1}{pq} \qquad  \textbf{(C)} \ \frac{p}{q^2} \qquad  \textbf{(D)}\ \frac{q}{p^2}\qquad \textbf{(E)}\ \frac{p}{q}$

Solution

By Vieta's Formulae, we have $\tan(\alpha)\tan(\beta)=q$ and $\cot(\alpha)\cot(\beta)=s$. Recalling that $\cot\theta=\frac{1}{\tan\theta}$, we have $\frac{1}{\tan(\alpha)\tan(\beta)}=\frac{1}{q}=s$.

Also by Vieta's Formulae, we have $\tan(\alpha)+\tan(\beta)=p$ and $\cot(\alpha)+\cot(\beta)=r$, and again using $\cot\theta=\frac{1}{\tan\theta}$, we have $\tan(\alpha)+\tan(\beta)=r(\tan(\alpha)\tan(\beta))$. Using $\tan(\alpha)\tan(\beta)=q$ and $\tan(\alpha)+\tan(\beta)=p$, we therefore deduce that $r=\frac{p}{q}$, which yields $rs = \frac{1}{q}\cdot\frac{p}{q}=\frac{p}{q^2}$.

Thus, the answer is $\boxed{\textbf{(C)} \ \frac{p}{q^2}}$.

See Also

1983 AHSME (ProblemsAnswer KeyResources)
Preceded by
Problem 19
Followed by
Problem 21
1 2 3 4 5 6 7 8 9 10 11 12 13 14 15 16 17 18 19 20 21 22 23 24 25 26 27 28 29 30
All AHSME Problems and Solutions


The problems on this page are copyrighted by the Mathematical Association of America's American Mathematics Competitions. AMC logo.png